¿Cómo probar que el cuanto N=4N=4{\cal N}=4 Super-Yang-Mills es superconforme?

Estoy especialmente interesado en pruebas ilustrativas elegantes que no impliquen muchos cálculos técnicos sencillos.

Además, ¿existe una prueba no perturbativa?

Considérelo como una teoría de N=1 super Yang-Mills con tres supercampos quirales adjuntos y aplique el análisis no perturbativo de Leigh-Strassler .

Respuestas (1)

En cualquier teoría supersimétrica, puede elegir que el acoplamiento de calibre sea el coeficiente de W α 2 en el superpotencial. Este acoplamiento de calibre funciona solo en un bucle, lo cual es una consecuencia fundamental de los teoremas de no renormalización. Los otros coeficientes de funcionamiento posibles son los términos cinéticos, Z ( m ) q q . Estos generalmente se vuelven a normalizar a todos los órdenes en la teoría de perturbaciones.

En norte = 4 el coeficiente de un bucle es cero. Esto es trivial (solo contando los campos). Por lo tanto, el acoplamiento de calibre (como se define anteriormente) no funciona. Pero norte = 4 relaciona las partículas de calibre con los supercampos quirales (todas las partículas de materia se sientan en una gran representación de norte = 4 ) y, por lo tanto, este último tampoco puede volver a normalizarse.

Este es un argumento ingenioso e intuitivo... Una lógica similar opera en muchos norte = 2 teorías también.

Buena respuesta. ¿Puede proporcionar una referencia por favor? ¿Entiendo correctamente que este argumento es solo perturbativo? Si es así, ¿hay alguna forma de extenderlo a uno no perturbador?
No es perturbador, porque el acoplamiento de calibre no funciona sin perturbaciones. No lo he visto explícitamente escrito en ninguna parte, pero estoy seguro de que (de lejos) no soy el primero que tuvo este pensamiento :)
Tu argumento es definitivamente mejor :)
Yuji: Creo que tal vez debas recurrir a este argumento de todos modos, incluso si lo haces con Leigh-Strassler. Esto se debe a que este último solo implica una línea unidimensional de CFT, pero no prueba que esta línea unidimensional coincida con la línea sobre la que se asienta N=4. Así que en algún momento tienes que invocar la simetría superior.
Bien, ¿puede proporcionar una referencia al teorema de no renormalización que está utilizando?
La página 96 en physics.uc.edu/~argyres/661/susy1996.pdf revisa el argumento. En general, puede probar que se trata de contribuciones de un bucle + no perturbativas. En N = 4, dado que no hay un bucle, no hay \Lambda y, por lo tanto, las contribuciones no perturbativas están ausentes.
Sí, eso es correcto. El sabor del argumento allí parece muy similar a lo que dije. Supongo que se puede aplicar un enfoque similar a las teorías en 3D o, de manera más general, a cualquier teoría en la que la noción de holomorfía tenga sentido.